Đến nội dung

Hình ảnh

Bất Đẳng Thức Qua Các Kỳ TS ĐH


  • Please log in to reply
Chủ đề này có 208 trả lời

#41
Khách- PiE_*

Khách- PiE_*
  • Khách
Nếu nói về cách khác thì có đấy , chẳng hạn cách Đạo hàm nè :
Giả sử $a=\max \{a,b,c \}$ .Xét hàm:
$ f(a)=\dfrac{1}{1+a^3}-\dfrac{3}{1+bc.a}$
Đạo hàm :
$ f'(a)=\dfrac{-3a^2}{(a^3+1)^2}+\dfrac{3bc}{(1+bc.a)^2}$
Khi đó:
$ f'(a) \geq 0 \Leftrightarrow \sqrt{bc}.(a^3+1) \geq a.(1+a.bc) \Leftrightarrow (a^2.\sqrt{bc}-1).(a-\sqrt{bc}) \geq 0$
Do $a,b,c$ là các số thực không nhỏ hơn 1 và $a=\max \{a,b,c \}$ nên ta suy ra hàm $f(a)$ đ�ồng biến trên $[1,+\infty]$
Từ đó ta đưa được bất đẳng thức về còn hai biến, và công việc tiếp theo là đơn giản( có thể đạo hàm tiếp hoặc biến đổi tương đương là được rùi ..:delta)

Bài viết đã được chỉnh sửa nội dung bởi dark templar: 31-05-2011 - 19:47


#42
supermember

supermember

    Đại úy

  • Hiệp sỹ
  • 1646 Bài viết

tui cũng có bài này nhưng ko phải là đề thi DH (đề thi của trường đấy!!)
cho a,b >0 ; a+b =1
CMR
$(a+\dfrac{1}{a})^2 + (b+\dfrac{1}{b})^2 \geq 12.5$

Bài này đơn giản rồi:áp dụng B.C.S ta có $(a+\dfrac{1}{a})^2 + (b+\dfrac{1}{b})^2 \geq \dfrac{(a+b+ \dfrac{1}{a}+\dfrac{1}{b})^2 }{2} \geq \dfrac{(1+4)^2}{2}= \dfrac{25}{2}$ (đpcm)

Bài viết đã được chỉnh sửa nội dung bởi inhtoan: 13-06-2009 - 08:42

Khi bạn là người yêu Toán, hãy chấp nhận rằng bạn sẽ buồn nhiều hơn vui :)

#43
daovantien34

daovantien34

    Binh nhì

  • Thành viên
  • 19 Bài viết
Bài 15: Cho a,b,c >0 t/m:$\dfrac{1}{a} + \dfrac{1}{b}+ \dfrac{1}{c} =1$
CMR:$ \dfrac{a^2}{a+bc} + \dfrac{b^2}{b+ac} + \dfrac{c^2}{c+ab} \geq \dfrac{a+b+c}{4}$

Bài viết đã được chỉnh sửa nội dung bởi Ispectorgadget: 19-01-2012 - 19:37


#44
tqnst

tqnst

    Trung sĩ

  • Thành viên
  • 170 Bài viết
1 lần nữa xin lỗi các bác nhé BĐT là thế này cơ:
$ \dfrac{a^4}{b+c} + \dfrac{b^4}{c+a} + \dfrac{c^4}{a+b} \geq \dfrac{a^3+b^3+c^3}{2} $

Bài viết đã được chỉnh sửa nội dung bởi dark templar: 31-05-2011 - 19:48


Trái tim anh, em Select bằng Mouse
Chốn hẹn hò: Forum - Internet
Lời yêu thương truyền bằng phương thức Get
Nhận dáng hình qua địa chỉ IP

Nếu một mai em vĩnh viễn ra đi
Anh sẽ chết giữa muôn ngàn biển Search
Lời tỏ tình không dễ gì Convert
Lưu ngàn đời vào biến Constant

Anh nghèo khó mang dòng máu Sun
Em quyền quý với họ Microsoft
Hai dòng Code không thể nào hoà hợp
Dẫu ngàn lần Debug em ơi

Sao không có một thế giới xa xôi
Linux cũng thế mà Windows cũng thế
Hai chúng ta chẳng thể nào chia rẽ
Run suốt đời trên mọi Platform.


#45
supermember

supermember

    Đại úy

  • Hiệp sỹ
  • 1646 Bài viết

1 lần nữa xin lỗi các bác nhé BĐT là thế này cơ:
$ \dfrac{a^4}{b+c} + \dfrac{b^4}{c+a} + \dfrac{c^4}{a+b} \geq \dfrac{a^3+b^3+c^3}{2} $

Bài này xài S.O.S có lẽ là được:BĐT $ \Leftrightarrow \sum \dfrac{a^3(a-b)+a^3(a-c)}{b+c} \geq 0$.Cái này đưa về S.O.S là okie

Bài viết đã được chỉnh sửa nội dung bởi inhtoan: 22-06-2009 - 15:11

Khi bạn là người yêu Toán, hãy chấp nhận rằng bạn sẽ buồn nhiều hơn vui :)

#46
supermember

supermember

    Đại úy

  • Hiệp sỹ
  • 1646 Bài viết

Cho a,b,c >0 t/m:
$\dfrac{1}{a} + \dfrac{1}{b}+ \dfrac{1}{c} =1$
CMR:$ \dfrac{a^2}{a+bc} + \dfrac{b^2}{b+ac} + \dfrac{c^2}{c+ab} \geq \dfrac{a+b+c}{4}$

Ta có a+b+c$ \leq 9$ nên ta c/m$ \sum \dfrac{a^2}{a+bc} \geq \dfrac{9}{4}$.Ta có ab+bc+ca=abc nên bc=abc-ab-ac nên ta c/m $ \sum \dfrac{a^2}{a-ab-ac+abc} \geq \dfrac{9}{4} \Leftrightarrow \dfrac{a}{(b-1)(c-1)} \geq \dfrac{9}{4}$.Đến đây thì quá đúng rồi.

Bài viết đã được chỉnh sửa nội dung bởi inhtoan: 13-06-2009 - 09:08

Khi bạn là người yêu Toán, hãy chấp nhận rằng bạn sẽ buồn nhiều hơn vui :)

#47
Sk8ter-boi

Sk8ter-boi

    (~.~)rubby(^.^)

  • Thành viên
  • 427 Bài viết

Ta có a+b+c$ \leq 9$ nên ta c/m$ \sum \dfrac{a^2}{a+bc} \geq \dfrac{9}{4}$.Ta có ab+bc+ca=abc nên bc=abc-ab-ac nên ta c/m $ \sum \dfrac{a^2}{a-ab-ac+abc} \geq \dfrac{9}{4} \Leftrightarrow \dfrac{a}{(b-1)(c-1)} \geq \dfrac{9}{4}$.Đến đây thì quá đúng rồi.

$1=\dfrac{1}{a}+ \dfrac{1}{b} + \dfrac{1}{c} \geq \dfrac{9}{a+b+c}$
nên a+b+c :Rightarrow 9

Bài viết đã được chỉnh sửa nội dung bởi inhtoan: 22-06-2009 - 15:10

i love 9C -- i luv u :x .... we'll never fall apart , but shine forever

9C - HN ams

#48
anhtuan_kt01

anhtuan_kt01

    Binh nhất

  • Thành viên
  • 27 Bài viết
[quote name='ducpbc' date='December 15, 2006 06:39 pm'] Bài này đơn giản rồi:áp dụng B.C.S ta có Hình đã gửi

Bài viết đã được chỉnh sửa nội dung bởi anhtuan_kt01: 17-12-2006 - 07:32

Cùng nhau phát triển forum này nhé !!!http://www.nxbgd.com.vn/toanhoctuoitre/
http://mathnfriend.net/http://toanthpt.net/

#49
dtdong91

dtdong91

    Tiến sĩ diễn đàn toán

  • Hiệp sỹ
  • 1791 Bài viết
Bài này có thể đặt [$a=\dfrac{1}{x},b=\dfrac{1}{y},c=\dfrac{1}{z}$ dể dk dễ hơn là $x+y+z=1$
Đến đây có thể d�ồng bậc hóa mà giải cho dễ

Bài viết đã được chỉnh sửa nội dung bởi dark templar: 31-05-2011 - 19:49

12A1-THPT PHAN BỘI CHÂU-TP VINH-NGHỆ AN

SẼ LUÔN LUÔN Ở BÊN BẠN

#50
duyptnk

duyptnk

    Lính mới

  • Thành viên
  • 6 Bài viết
schawrz đc ko nhỉ ^^

Bài viết đã được chỉnh sửa nội dung bởi duyptnk: 17-12-2006 - 11:47

Trời xanh đã trót sanh ra Lượng
Đất mẹ sao còn nảy Chu lang ...

#51
supermember

supermember

    Đại úy

  • Hiệp sỹ
  • 1646 Bài viết

$1=\dfrac{1}{a}+ \dfrac{1}{b} + \dfrac{1}{c} \geq \dfrac{9}{a+b+c}$
nên $a+b+c \geq 9$

Ừ,đúng r�ồi,đoạn này anh quên khuấy đi.Nhưng vẫn có thể c/m được bằng cách c/m $ \sum \dfrac{a}{(b-1)(c-1)} \geq \dfrac{a+b+c}{4}$.Cái này thì quy đ�ồng lên là okie

Bài viết đã được chỉnh sửa nội dung bởi dark templar: 31-05-2011 - 19:49

Khi bạn là người yêu Toán, hãy chấp nhận rằng bạn sẽ buồn nhiều hơn vui :)

#52
Khách- PiE_*

Khách- PiE_*
  • Khách
He he ... Bài này mình có hai cách vô cùng dễ hiểu.
Lời giải 1. Do vai trò đối xứng của $(a,b,c)$ và $ (\dfrac{a}{a+bc} ,\dfrac{b}{b+ca},\dfrac{c}{c+ab})$ là cùng giảm . Thế thì áp dụng BDT Trê-bư_sép ta có:
$ M= a.\dfrac{a}{a+bc} +b.\dfrac{b}{b+ca}+c.\dfrac{c}{c+ab} \geq \dfrac{a+b+c}{3}.(\dfrac{a}{a+bc} +\dfrac{b}{b+ca}+\dfrac{c}{c+ab})$
Rùi đặt : N=$ \dfrac{a}{a+bc} +\dfrac{b}{b+ca}+\dfrac{c}{c+ab}$
Chú ý rằng từ điều kiện :$ \dfrac{1}{a}+ \dfrac{1}{b} +\dfrac{1}{c} =1 $ ta suy ra $ abc=ab+bc+ca $ .
Bây giờ áp dụng bất đẳng thức Cauchy-Shwart tá có:
N=$ \dfrac{a^2}{a^2+abc} +\dfrac{b^2}{b^2+abc}+\dfrac{c^2}{c^2+abc} \geq \dfrac{(a+b+c)^2}{a^2+b^2+c^2+3abc}=\dfrac{(a+b+c)^2}{a^2+b^2+c^2+3(ab+bc+ca)}=\dfrac{(a+b+c)^2}{(a+b+c)^2+(ab+bc+ca)} \geq \dfrac{(a+b+c)^2}{(a+b+c)^2+\dfrac{(a+b+c)^2}{3}}=\dfrac{3}{4} $
Từ đó ta có bdt cần chứng minh.Dấu bằng khi $ a=b=c $ .

đó là lời giải đầu , còn sau đây cơ bản hơn .Chẳng cần sử dụng Định lí nào cả .
Lời giải 2. :Từ $ \dfrac{1}{a}+ \dfrac{1}{b} +\dfrac{1}{c} =1 $ ta suy ra $ bc=\dfrac{a.(b+c)}{a-1} $ .Do đó :
$ \dfrac{a^2}{a+bc}=\dfrac{a^2}{a+\dfrac{a.(b+c)}{a-1}}=\dfrac{a}{1+\dfrac{b+c}{a-1}}=\dfrac{a.(a-1)}{a+b+c-1}$
Tương tự ta có :
$ S= \dfrac{a^2}{a+bc} +\dfrac{b^2}{b+ca}+\dfrac{c^2}{c+ab} =\dfrac{a.(a-1)}{a+b+c-1}+\dfrac{b.(b-1)}{a+b+c-1}+\dfrac{c.(c-1)}{a+b+c-1}= \dfrac{a.(a-1)+b.(b-1)+c.(c-1)}{a+b+c-1}=\dfrac{M^2-M-2N}{M-1}$
Trong đó :$ M=a+b+c $ , và $ N=abc=ab+bc+ca $.
Vậy thì :
$ S \geq \dfrac{a+b+c}{4}=\dfrac{M}{4}$ :D $3.M^2-3.M \geq 8.N $ .
Chú ý :
*$ N=ab+bc+ca \leq \dfrac{(a+b+c)^2}{3}= \dfrac{M^2}{3}$
*$M= (a+b+c).(\dfrac{1}{a}+ \dfrac{1}{b} +\dfrac{1}{c}) \geq 9 $ :D $ M \geq 9 $
Do đó :$3.M^2-3.M =8.\dfrac{M^2}{3}+\dfrac{M.(M-9)}{3}\geq 8.N $ .

Vậy ta suy ra bdt cần chứng minh .

hai lời giải cũng khá hay đấy chứ ???? hehe
:geq :D

Bài viết đã được chỉnh sửa nội dung bởi inhtoan: 02-10-2009 - 10:11


#53
anhtuan_kt01

anhtuan_kt01

    Binh nhất

  • Thành viên
  • 27 Bài viết
bất đẳng thức hình học thì sao nhỉ :D
:)
bài 1
cho $S=\dfrac{3}{2} ,a,b,c$ lần lượt là độ dài các cạnh $BC ,AC,AB$ và $h_a,h_b,h_c$ tương ứng là độ dài các dường cao hạ từ đỉnh ABC của $\Delta ABC$.CMR
$\left( \dfrac{1}{a} +\dfrac{1}{b} +\dfrac{1}{c} \right) \left(\dfrac{1}{ha} + \dfrac{1}{hb} +\dfrac{1}{c} \right) \geq 3$--------(<span style='color:blue'>đề thi DB khối D-2002)</span>
bài 2
cho tu diện ABCD có $AD \perp (ABC)$ và $\Delta ABC$ vuông tại A $AD=a,AC=b,AB=c$
CMR
$2S \geq \sqrt{abc(a+b+c)}$ ( với S dt $\Delta BCD$)
<span style='color:blue'>đề thi DB khối D---2003</span>
bài 3
cho tứ diện OABC có 3 cạnh OA,OB,OCđội một vuông góc
gọi $\alpha , \beta, \gamma$ lần lượt là các góc giữa $\mp{(ABC)}$ với các $\mp{(OBC)};(OCA);(OAB)$ CMR :
$\cos{ \alpha} +\cos {\beta} + \cos{\gamma} \leq \sqrt{3}$
<span style='color:blue'>đề thi dự bị khối A-2002</span>

Bài viết đã được chỉnh sửa nội dung bởi dark templar: 01-06-2011 - 08:37
Latex

Cùng nhau phát triển forum này nhé !!!http://www.nxbgd.com.vn/toanhoctuoitre/
http://mathnfriend.net/http://toanthpt.net/

#54
Nguyễn Thành An

Nguyễn Thành An

    Lính mới

  • Thành viên
  • 1 Bài viết
ra được chết liên giải lại đi bồ ơi nếu dùng bdt tri tuyệt đối không ra được

#55
vermissa

vermissa

    Binh nhất

  • Thành viên
  • 27 Bài viết
Bài 17:Cho $x+y+z=1$
Tìm min của $\dfrac{1}{x}+\dfrac{1}{y}+\dfrac{1}{z}$

Bài viết đã được chỉnh sửa nội dung bởi Ispectorgadget: 19-01-2012 - 19:37
Latex


#56
vermissa

vermissa

    Binh nhất

  • Thành viên
  • 27 Bài viết
Bài 18: Cho $x+y+z=1$
Tìm min của $\left( \dfrac{1}{x}-1 \right)\left( \dfrac{1}{y}-1 \right)\left( \dfrac{1}{z}-1 \right)$

Bài viết đã được chỉnh sửa nội dung bởi Ispectorgadget: 19-01-2012 - 19:38
Latex


#57
supermember

supermember

    Đại úy

  • Hiệp sỹ
  • 1646 Bài viết
Bài này đơn giản thôi,chỉ cần quy đ�ồng lên hết ta sẽ có $ \dfrac{(a+b)(b+c)(c+a)}{abc} \geq 8$.Tất nhiên với $a,b,c>0$

Bài viết đã được chỉnh sửa nội dung bởi dark templar: 01-06-2011 - 08:40

Khi bạn là người yêu Toán, hãy chấp nhận rằng bạn sẽ buồn nhiều hơn vui :)

#58
NPKhánh

NPKhánh

    Tiến sĩ toán

  • Thành viên
  • 1115 Bài viết
Bài 19: CMR: nếu x;y;z là các số thực dương thỏa mãn điều kiện $xy + yz +zx = xyz$ thì $\Large \dfrac{1}{x +2y+3z} + \dfrac{1}{2x + 3y +z} + \dfrac{1}{3x + y + 2z} <\dfrac{3}{16} $

Bài viết đã được chỉnh sửa nội dung bởi Ispectorgadget: 19-01-2012 - 19:38

http://mathsvn.violet.vn trang ebooks tổng hợp miễn phí , nhiều tài liệu ôn thi Đại học



http://www.maths.vn Diễn đàn tổng hợp toán -lý - hóa ... dành cho học sinh THCS ;THPT và Sinh viên


#59
supermember

supermember

    Đại úy

  • Hiệp sỹ
  • 1646 Bài viết

CMR: nếu x;y;z là các số thực dương thỏa mãn điều kiện $xy + yz +zx = xyz$ thì $\Large \dfrac{1}{x +2y+3z} + \dfrac{1}{2x + 3y +z} + \dfrac{1}{3x + y + 2z} <\dfrac{3}{16} $

Áp dụng ngay cái này:$\dfrac{36}{x+y+y+z+z+z} \leq \dfrac{1}{x}+\dfrac{2}{y}+\dfrac{3}{z} $ với $\dfrac{1}{x}+\dfrac{1}{y}+\dfrac{1}{z}=1 $

Bài viết đã được chỉnh sửa nội dung bởi dark templar: 01-06-2011 - 08:41

Khi bạn là người yêu Toán, hãy chấp nhận rằng bạn sẽ buồn nhiều hơn vui :)

#60
raynalhunter

raynalhunter

    Lính mới

  • Thành viên
  • 4 Bài viết
Bài 20: Một bài khá khó trong báo toán giúp e với!
Cho $a,b,c \in \left[\dfrac{1}{3} ;3 \right]$
CMR
$ \dfrac{ a}{a+b} + \dfrac{b}{b+c} +\dfrac{ c}{c+a} \geq \dfrac{7}{5}$

Bài viết đã được chỉnh sửa nội dung bởi Ispectorgadget: 19-01-2012 - 19:38
Latex

It takes 2 to tango,2 to kiss,2 to talk & remenisce,so many good things come in 2,& one of those things is me & u!




2 người đang xem chủ đề

0 thành viên, 2 khách, 0 thành viên ẩn danh